Improper integral comparison test

Click For Summary
The discussion centers on determining the convergence or divergence of the improper integral ∫_{0}^{∞} |sin(x)|/x² dx. Initial comparisons with 1/x² and sin(x)/x² were attempted, but the divergence of 1/x² at the lower limit complicates the analysis. A suggestion is made to split the integral into two parts: from 0 to π/2 and from π/2 to infinity, allowing for separate evaluations. In the interval [0, π/2], establishing an inequality between |sin(x)| and a non-periodic function like f(x) = x is recommended for further analysis. The discussion highlights the need for careful handling of the integral's limits and appropriate comparisons to draw conclusions about convergence.
hitemup
Messages
81
Reaction score
2
Member warned about not using the homework template
The question asks whether the following converges or diverges.

\int_{0}^{\infty } \frac{\left | sinx \right |}{x^2} dx

Now I think there might be a trick with the domain of sine function but I couldn't make up my mind on this.
I tried to compare it with 1/x^2, (sinx)/x, and sinx. I actually expected that I would get something good with 1/x^2, but as the lower limit of the integral is zero, it ended up with infinity on (0, inf) and since 1/x^2 is greater than (sinx)/x^2, and is divergent as we just found, we cannot say whether the given function diverges or converges. So I'm wondering what is the right track on this question?
 
Last edited:
Physics news on Phys.org
\int_{0}^{\infty } \frac{\left | sinx \right |}{x^2} dx =

\int_{0}^{ \frac{\pi}{2} } \frac{\left | sinx \right |}{x^2} dx + \int_{ \frac{\pi}{2} }^{\infty} \frac{\left | sinx \right |}{x^2} dx

Deal with the two integrals separately.

In the interval [0,\frac{\pi}{2}] you can establish an inequality between |\sin(x)| and a non-periodic function. Try comparing it to f(x) = x.
 
Question: A clock's minute hand has length 4 and its hour hand has length 3. What is the distance between the tips at the moment when it is increasing most rapidly?(Putnam Exam Question) Answer: Making assumption that both the hands moves at constant angular velocities, the answer is ## \sqrt{7} .## But don't you think this assumption is somewhat doubtful and wrong?

Similar threads

  • · Replies 5 ·
Replies
5
Views
2K
  • · Replies 4 ·
Replies
4
Views
1K
  • · Replies 3 ·
Replies
3
Views
1K
  • · Replies 16 ·
Replies
16
Views
4K
  • · Replies 4 ·
Replies
4
Views
1K
  • · Replies 2 ·
Replies
2
Views
2K
  • · Replies 2 ·
Replies
2
Views
2K
  • · Replies 6 ·
Replies
6
Views
2K
Replies
3
Views
2K
  • · Replies 6 ·
Replies
6
Views
11K